Points S, U, and T are the midpoints of the sides of TrianglePQR.

Triangle S U T is inside of triangle P Q R. Points S, U, and T are the midpoints of triangle P Q R.

Which statements are correct? Check all that apply.
One-halfQP = UT
One-halfTS = RQ
SU = PR
SU ∥ RP
UT ⊥ RP

Answers

Answer 1

Option A and D. The statements that are correct here are

1/2 QP = UTSU ║ RP

How to arrive at the statements about the midpoints

If the two points UT and QP are parallel, then UT is going to be the half of QP. This would be due to a scale factor of 1/2.

Then at a midpoint T is that of PR. SU is parallel to RP. SU is half of RP due to as scale factor of 1/2.

Read more on triangles here:

https://brainly.com/question/2644832

#SPJ1

Answer 2

Answer: A and D E2020

Step-by-step explanation:

E2020 geometry B!! :)


Related Questions

Need help with this question

Answers

As per definition:Complementary angles are adjacent angles that sum to 90°;Supplementary angles are adjacent angles that sum to 180°.

Now, ORQ is a straight angle and RS its bisector.

It means:

ORS = QRS = 180°/2 = 90°

So, ∠ORS and ∠QRS are supplementary.

We also have ray RP, that divides the angles ORQ and QRS.

It gives us one more pair of supplementary angles:

∠ORP and ∠QRP.

Since ∠QRS is right angle, its parts are forming a pair of complementary angles:

∠SRP and ∠QRP are complementary

The total cost, C, for running an advertisement in a local newspaper, The Free Press, is made up of an initial cost of $12 plus a charge of $5 per day. A rival newspaper, The Banner, is currently running a special on advertisements at $8 per day with no initial cost.
a) Write an equation representing the cost in The Free Press.
b) Write an equation representing the cost in The Banner.
c) For each newspaper, create a table of values.
d) Use Geometer’s Sketchpad to graph each cost on the same set of coordinate axes.
e) Which newspaper would you use for an ad that ran 1 day?
f) Which newspaper would you use for an ad that ran 12 days?

Answers

The equation representing the cost in The Free Press is 12 + 5d.

How to depict the equation?

The equation representing the cost in The Free Press will be:

= 12 + (5 × d)

= 12 + 5d

The equation representing the cost in The Banner will be:

= 8 × d

= 8d

The table of value for Free Press will be:

1st day = 12

2nd day = 17

3rd day = 22

4th day = 27

5th day = 32

The table of value for the Banner will be:

1st day = 8

2nd day = 16

3rd day = 24

4th day = 32

5th day = 40

The newspaper that will be used to run the advertisement for the first day is Free Press since it's cheaper.

The newspaper that will be used to run the advertisement for twelve days will be:

Free press = 12 + 5d

= 12 + 5(12)

= 12 + 60 = 72

The banner = 8d = 8 × 12 = 96

Therefore, for 12 days, Free Press is cheaper.

Learn more about equations on:

brainly.com/question/2972832

#SPJ1

The 8. is just the number of the question.

Answers

The value of (1+tanx)(1-tanx)+sec²x is 0 and the value of [tex]\frac{sec^{2}-4 }{sec+2}[/tex] is [tex]sec\theta \ -2[/tex].

Given that, (1+tanx)(1-tanx)+sec²x.

Now, (1-tan²x)+sec²x

=1-tan²x+sec²x

=1+sec²x-tan²x

=1-1=0

Now, evaluate [tex]\frac{sec^{2}-4 }{sec+2}[/tex]

[tex]=\frac{sec^{2} \theta \ -2^{2} }{sec\theta \ +2} =\frac{(sec \theta \ +2)(sec \theta \ -2)}{sec \theta \ +2}[/tex] (∵a²-b²=(a+b)(a-b))

[tex]=sec \theta \ -2[/tex]

Hence, the value of (1+tanx)(1-tanx)+sec²x is 0 and the value of [tex]\frac{sec^{2}-4 }{sec+2}[/tex] is [tex]sec\theta \ -2[/tex].

To learn more about trigonometric identities visit:

https://brainly.com/question/12537661.

#SPJ1

A motor bike depreciated in value from 4500 to 3285. calculate the percentage depreciation

Answers

The percentage depreciation is 27 % .

What is percentage ?

Rather than being expressed as a fraction, a percentage is a piece of a whole expressed as a number between 0 and 100. Nothing is zero percent; everything is 100 percent; half of something is 50 percent; and nothing is zero percent. You divide the part of the whole by the whole and multiply the result by 100 to get the percentage.

Given ,

         from 4500 to 3285 is depreciation value of a motor bike.

So,

         =      [tex]\frac{( 4500 - 3285 )} {4500} * 100[/tex]

         =       [tex]\frac{1215}{4500} * 100\\[/tex]

           = [tex]\frac{121500}{4500}[/tex]

          = 27 %

Therefore, The percentage depreciation is 27 % .

Learn more about percentage depreciation brainly.com/question/13894858

#SPJ4

           

hep me out with this question i will give you brainly!

Answers

Answer:

c because have great day

A because it’s written in a full sentence format.

A pet supply store lets you donate your change to the local animal shelter. Denise buys a bag of dog food for $15.74 and a squeaky toy for $3.98. She gives the cashier $20. If she gives all of her change, how much will Denise donate to the animal shelter?

Answers

Considering her change, it is found that she will donate $0.28 to the animal shelter.

How to find the amount of change in a purchase?

The amount of change is given by the subtraction of the total paid and the price of the products.

In this problem, the price of the products bought is given by:

$15.74 + $3.98 = $19.72

She paid $20, hence the change is given by:

$20 - $19.72 = $0.28.

More can be learned about the amount of change in a purchase at https://brainly.com/question/10866253

#SPJ1

Find x and y.
x - y = 2
3x - y = 12​

Answers

the answer is

X=5

y=3

hope you like it

Find the midpoint of a A(3, -1) and B(x, y) is P(4, - 5). Find the coordinates of B. pls answer it is urgent i will mark him or her brainlest​

Answers

Answer:

is this tye required answer

Just an easy question for 100 points! :)

Answers

If the function is translated 4 units down, the resulting function will be

y = x - 4 while the vertical stretch by 3units will be y = 3(x-4)

Transformation of functions

Transformation of function is a way of changing the position of a line or curve on an xy place.

Given the equation of a line as shown;

y = x (This will serve as the parent function)

If the function is translated 4 units down, the resulting function will be

y = x - 4. If the resulting function is stretched by a factor of 3, the final function will be y = 3(x - 4)

Learn more on translation here: https://brainly.com/question/12861087

#SPJ1

Here is a list of ages (years) of children in a room: 9 , 9 , 9 , 4 , 9 , 1 , 10 State the median.

Answers

Answer:

the median is 9

Explanation

Finding the median is easy, all you need to do is put the numbers in numeric order and the one that's in the middle of the numbers is the median

9 , 9 , 9 , 4 , 9 , 1 , 10  =   1, 4, 9, (9), 9, 9, 10

                                              (median)

SOLVING

[tex]\Large\maltese\underline{\textsf{A. What is Asked}}[/tex]

Given the Ages of students: 9,9,9,4,9,1,10. State the median

[tex]\Large\maltese\underline{\textsf{B. This problem has been solved!}}[/tex]

[tex]\boxed{\begin{minipage}{7cm} Before finding the median, we\\ need to arrange \\ our data from least to greatest. \end{minipage}}[/tex]

Thus,

[tex]\bf{1,4,9,9,9,9,10}[/tex].

[tex]\fbox{The median is the number in the middle.}[/tex]

Thus,

[tex]\bf{9}[/tex]

[tex]\cline{1-2}[/tex]

[tex]\bf{Result:}[/tex]

            [tex]\bf{=9}[/tex]

Extra information [tex]\curvearrowright[/tex]

[tex]\begin{tabular}{c | 1} Data Parameter & What to Do \\ \cline{1-2} \\ Mean & Add all the numbers and divide by how many there are \\ Median & Arrange the numbers from least to greatest and take the number \\ & in the middle. Take the mean of the 2 numbers in the middle \\ & if there's an even amount of numbers \\ Mode & The number that occurs the most \\ Range & The largest number minus the smallest number \\ Minimum & The smallest value \\ Maximum & The largest value \end{tabular}[/tex]

[tex]\LARGE\boxed{\bf{aesthetic\not1\theta\ell}}[/tex]

hellp help
x =x=x, equals

^\circ



degrees

Answers

The sum of the unknown angle and 33 degrees are supplementary. The measure of x is 147 degrees

Supplementary angles

The sum of the unknown angle and 33 degrees are supplementary to have:

x + 33 = 180

Subtract 33 from both sides

x + 33 - 33 = 180 - 33

x = 180 - 33

x = 147degrees

Hence the measure of x is 147 degrees

Learn more on supplementary angles here: https://brainly.com/question/12919120

#SPJ1

Answer:

Step-by-step explanation:

x + 33° = 180°

reason :( they are cointerior angle )

x = 180° - 33°

x = 147°

Use the given graph to determine the limit, if it exists. (4 points)
A coordinate graph is shown with a horizontal line crossing the y axis at three that ends at the open point 2, 3, a closed point at 2, 1, and another horizontal line starting at the open point 2, -2.
Find limit as x approaches two from the left of f of x. and limit as x approaches two from the right of f of x. .

Does not exist; does not exist
1; 1
-2; 3
3; -2

Answers

Limit of x approaches two from the left of f of x: 3. Limit of x approaches two from the right of f of x: - 1 (negative 1)

How to find the value of the function as x approaches infinity (+ve or -ve)?

If limits exist, we can take limits of the function, where x tends to -∞ or ∞, and that limiting value will be the value the function will approach.

In order to find the limit as x approaches two from the left of f of x:

Lim x→2- f(x) = ?

According to the graph, when x approaches two from the left (x<2), the function approaches 3:

Lim x→2- f(x) = 3

IN order to find the limit as x approaches two from the right of f of x:

Lim x→2+ f(x) = ?

According to the graph, when x approaches two from the right (x>3), the function approaches -1:

Lim x→2+ f(x) = - 1

Learn more about one-sided limits here:

https://brainly.com/question/23625942

#SPJ1

Joe paid $14 for a board game. this is 70% of the original price. what was the original price?

Answers

Step-by-step explanation:

It is given that Joe paid 14.00 for a board game this is 70% of the original price what is the original price.

Let the original price of board game be

X.

x * 70/100 = 14

x * 7/10 = 14

Multiply both sides by 10.

7x = 140

Divide both sides by 7.

x = 20

Which three pairs of measurements are possible side lengths for the triangle?
A
B
60°
F
AB=4, BC= 4√3
OBC= 2√3, AC=2
AB=3, AC=3√3
BC=10, AC= 4√3
AB=7, AC=14
AB=11, BC=11√3
30°
C

Answers

Answer:

Options (1), (5), and (6)

Step-by-step explanation:

See attached image

Three pairs of measurements which are possible side lengths for the triangle are,

1.  AB = 4, BC = 4√3

5 AB = 7, AC = 14.

6. AB = 11, BC = 11√3

What is mean by Triangle?

A triangle is a three sided polygon, which has three vertices and three angles which has the sum 180 degrees.

Now, We get;

The sides of this triangles has lengths in the ratio x : √3x : 2x

where 2x is the hypotenuse.

Now, When C = 30,  the side AB is the small leg (=x).

When B = 90, the hypotenuse is AC ( =2x).

When A =  60, the larger leg is BC (=√3x).

1.  AB = 4, BC = 4√3 :- Are possible legs.

5 AB = 7, AC = 14. :-  AB = small leg, AC = hypotenuse.

6. AB = 11, BC = 11√3:-   AB = small, BC = larger leg.

Learn more about the triangle visit;

brainly.com/question/1058720

#SPJ5

An adult gerbil at Fish 'n' weighed 1/5 lb. A young gerbil weighed 1/4 of that amount. How many did the young gerbil weigh?

Answers

Weight of young gerbil: 1/5*1/4 = 1/20 lbs

Find the answer

a
b
c
d

Answers

Answer: a

Step-by-step explanation:

[tex]\left(\frac{f}{g} \right)(x)=\frac{f(x)}{g(x)}=\frac{\sqrt{x^{2}-1}}{\sqrt{x-1}}}=\sqrt{\frac{x^{2}-1}{x-1}}=\sqrt{\frac{(x-1)(x+1)}{x-1}}=\boxed{\sqrt{x+1}}[/tex]

100 POINTS!!!
PLEASE HELP ITS EZ :)

Answers

In 2.44 seconds lindsey be 30 feet in the air.

What is Quadratic Equation?

A quadratic equation is a second-order polynomial equation in a single variable x, ax²+bx+c=0. with a ≠ 0 .

Given:

y= -16x²+33x+45

If the height is 30 feet.

30= -16x²+33x+45

-16x²+33x+45-30=0

-16x²+33x+15=0

16x²-33x-15=0

Solving the quadratic equation

x=-b±√b²-4ac/ 2a

x= 33 ± √2049/32

x= (33+ √2049)/32    and x=33-√2049/32

x=2.44 and -0.3833

Learn more quadratic equation here:

https://brainly.com/question/2263981

#SPJ1

Solve the following system of equations using elimination by eliminating the y variable
first.
5x + 2y = 3
2x + 3y = 10

Answers

The solution to the system of equations is x = -1 and y = 4

How to solve the system of equations?

The system of equations is given as:

5x + 2y = 3

2x + 3y = 10

Multiply the first equation by 2 and the second by 5.

10x + 4y = 6

10x + 15y= 50

Subtract the above equations

10x - 10x + 4y - 15y = 6 - 50

This gives

-11y = -44

Divide by -11

y = 4

Substitute y = 4 in 5x + 2y = 3

5x + 2 * 4 = 3

This gives

5x + 8 = 3

Subtract 8 from both sides

5x = -5

Divide by 5

x = -1

Hence, the solution to the system of equations is x = -1 and y = 4

Read more about system of equations at:

https://brainly.com/question/12895249

#SPJ1

What is the product of the...

Answers

Step-by-step explanation:

the answer is C) 8x^5y - 6x^4y^2

pls like and follow.

hope you understand as well

Which is an equation of the line through (0,0) and (2,-3)?

Answers

Answer:

y = -1.5x

How to build equation?

Given points: (0, 0), (2, -3)

Find slope:

[tex]\sf slope: \dfrac{y_2 - y_1}{x_2- x_1} \ \ where \ (x_1 , \ y_1), ( x_2 , \ y_2) \ are \ points[/tex]

[tex]\rightarrow \sf slope \ (m) = \dfrac{-3-0}{2-0} = -1.5[/tex]

Then find equation:

[tex]y-y_1 = m(x - x_1)[/tex]

[tex]y - 0 = -1.5(x -0)[/tex]

[tex]y = -1.5x[/tex]

Answer:

y = (-3/2)x

Step-by-step explanation:

I am going to answer this question in slope-intercept form, which is y = mx + b (m is the slope, b is the y-intercept). The question already gives us the y-intercept, or the point at which the graph crosses the y-axis. It is (0, 0). Now, all we have to do is find the slope, which we can do using the following formula:

[tex]m=\frac{y_{2}-y_{1} }{x_{2}-x_{1}}[/tex]

In this formula, (x1, y1) and (x2, y2) are two points that are given to us [the first point is (0,0) and the second point is (2,-3). We can solve for the slope by plugging in:

[tex]m=\frac{-3-0}{2-0}=\frac{-3}{2}[/tex]

Now that we have m and b, we can plug into slope-intercept form to get our equation: y = (-3/2)x + 0 or y = (-3/2)x


4. In how many ways can a team of 3 boys and 3 girls be selected from 5 boys and 4 girls?

Answers

A team of 3 boys and 3 girls be selected from 5 boys and 4 girls in 40 ways

How to determine the number of ways?

The selections are given as:

3 boys from 5 boys

3 girls from 4 girls

Each selection is calculated using:

[tex]^nC_r = \frac{n!}{(n- r)!r!}[/tex]

So, we have:

[tex]Ways = ^5C_3 * ^4C_3[/tex]

Apply the combination formula

[tex]Ways = \frac{5!}{2!3!} * \frac{4!}{1!3!}[/tex]

Expand

[tex]Ways = \frac{5 * 4 * 3!}{2 * 3!} * \frac{4 * 3!}{1 * 3!}[/tex]

Simplify

Ways = 5 * 2 * 4

Evaluate

Ways =40

Hence, the number of ways is 40

Read more about combination at:

https://brainly.com/question/11732255

#SPJ1

A projectile is fired from the ground with an upward speed of 245 m/s. What is the maximum height of the projectile?

Answers

The maximum height of the projectile fired from the ground with an upward speed of 245 m/s is 3001.25 m

What is an equation?

An equation is an expression that shows the relationship between two or more variables and numbers.

The maximum height (H) of a projectile is given as:

H = u²/2g

Where u is the initial speed = 145 m/s, g is the acceleration due to gravity = 10 m/s²

H = 245²/ 2(10) = 3001.25 m

The maximum height of the projectile fired from the ground with an upward speed of 245 m/s is 3001.25 m

Find out more on equation at: https://brainly.com/question/2972832

#SPJ1

11. (21x² - 6x +14) + (x²-3x+18)

Answers

Answer:

22x^2-9x+32

Step-by-step explanation:

(21x² - 6x +14) + (x²-3x+18)

To add, we need to combine like terms

21x^2+x^2     -6x-3x    +14+18

22x^2-9x+32

Answer:

22x² - 9x + 32

Step-by-step explanation:

(21x² - 6x +14) + (x²-3x+18)

Get rid of the parenthesis:

21x² - 6x + 14 + x² - 3x + 18

Group like terms:

22x² - 9x + 32

true or false: prostulates are accepted as true without proof

Answers

Answer:

true

Step-by-step explanation:

Since their wise or intelligent guess

How many solutions exist for the system of equations graphed below?

Answers

Answer: one

Step-by-step explanation:

     When a system of equations is graphed the solutions are the point of intersections. Here, we only have one point of intersection since these are both linear equations. That means there is only one solution.

the question is on the image​

Answers

Answer:

nth term: 0.5n² + 2.5n + 3

Explanation:

pattern 1 = 6 bowls

pattern 2 = 10 bowls

pattern 3 = 15 bowls

pattern 4 = 21 bowls

First difference:

10 - 6 = 415 - 10 = 521 - 15 = 6

Second difference:

5 - 4 = 16 - 5 = 1

This is not an arithmetic nor a geometric sequence.

Solve:

2a  = 2nd difference

2a = 1

a = 1/2 or 0.5

===================

3a + b = 1st difference

3a + b = 4

b = 4 - 3(1/2)

b = 2.5

====================

a + b + c = first term

1/2 + 2.5 + c = 6

c = 3

Quadratic nth term:

0.5n² + 2.5n + 3

How can you quickly determine the number of roots a polynomial will have by looking at the equation?

Answers

One can determine the number of roots by seeing the degree of the given polynomial.

The number of roots can be determined by just seeing the highest power of the given equation.

1) for the equation  x-4=0

here, the highest power of the equation is one. So, it will have one root.

What is the polynomial?

A polynomial is an expression consisting of indeterminates and coefficients, that involves only the operations of addition, subtract multiplication, on,  and non-negative integer exponentiation of variables.

Let's check it by simplifying x-4=0  which implies x=4

Hence, the equation has only one root namely x = 4.

2) Consider a quadratic equation,

10t^2-t-3=0

here, the highest power of the equation is two. So, it will have two roots.

Let's check it by simplifying using the middle term splitting method,

10t^2+5t-6t-3=0

5t(2t+1)-3(2t+1)=0

(5t-3)(2t+1)=0

t=-3/5 or t=-1/2

Thus, the equation has two roots.

Hence, one can determine the number of roots by seeing the degree of the given polynomial.

To learn more about the polynomial visit:

https://brainly.com/question/2833285

#SPJ1

what is inversion transformation ​

Answers

Answer:

Inversión en una circunferencia es una transformación que conserva la magnitud de los ángulos pero cada ángulo se transforma en un ángulo de signo opuesto . El sentido o dirección de cada ángulo se invierte. La inversión en una circunferencia es una transformación anti-conforme(como la reflexión en una recta).

Step-by-step explanation:

El sentido o dirección de cada ángulo se invierte. La inversión en una circunferencia es una transformación anti-conforme(como la reflexión en una recta).

Answer:

inversion transformations are a natural extension of Poincaré transformations to include all conformal one-to-one transformations on coordinate space-time.

You are able to walk uphill at a speed of 2 miles per hour and downhill at a speed of 3 miles per hour. What does the graph represent?

Answers

Using a piece-wise function, the representation of the graph is given as follows:

Walking uphill for 3 hours and downhill for 1 hour.

What is a piece-wise function?

A piece-wise function is a function that has multiple definitions, depending on the input.

In this problem, we have that:

For the first 3 hours, the slope is of 2 miles per hour, equivalent to walking uphill.For the final hour, the slope is of 3 miles per hour, equivalent to walking downhill.

Hence the meaning of the graph is:

Walking uphill for 3 hours and downhill for 1 hour.

More can be learned about piece-wise functions at https://brainly.com/question/27262465

#SPJ1

if you can solve, i will give brainliest!
What is the sum of StartRoot 3 EndRoot – i and 2 – StartRoot 3 EndRooti?

(2 + StartRoot 3 EndRoot) – (1 –StartRoot 3 EndRoot )i
(2 + StartRoot 3 EndRoot) – (1 + StartRoot 3 EndRoot)i
(2 + StartRoot 3 EndRoot) + (1 + StartRoot 3 EndRoot)i
(2 + StartRoot 3 EndRoot) + (1 – StartRoot 3 EndRoot)i

Answers

Answer:

(2 + StartRoot 3 EndRoot) – (1 + StartRoot 3 EndRoot)i

Step-by-step explanation:

I'm 100% sure! ^^

Answer:

B: (2 + StartRoot 3 EndRoot) – (1 + StartRoot 3 EndRoot)i

Step-by-step explanation:

Took the quiz

Other Questions
Determine the composition of transformations that would map figure ABCD to figure A''B'C''D' Suppose that consumers decide to walk to work more frequently and drive cars less. Companies that make walking shoes hire workers, while automobile companies lay off workers. This is an example of What should for extraction of protein from placental tissue by some advance tequnique FIND R, Interest problem Argentina can produce 150 pounds of beef or 20 cars; in contrast the United States can produce 90 pounds of beef or 80 cars. Which country has the absolute advantage in producing beef The number line below is correctly represented by which of the following intervals? In an experiment carried out to study the energy flow through an ecosystem, scientists measured the total solar energy captured by plants to be 10,000 kilocalories. The portion of this available to the next trophic level will be ___. The principal represents an amount of money deposited in a savings account subject tocompound interest at the given rate.A. Find how much money there will be in the account after the given number of years.B. Find the interest earned.Click the icon to view some finance formulas.A. The amount of money in the account after 5 years is $(Round to the nearest hundredth as needed.)Principal$11,000Rate5%CompoundedannuallyTime5 years When a party has its members vote unofficially to help determine who would win a primary, it is conducting a _______ poll. Musical representation of shepherds with a bagpipe-like drone is an example of what compositional technique For the system of inequalities below, which ordered pair is a solution?3x-y A food truck did a daily survey of customers to find their food preferences. the data is partially entered in the frequency table. complete the table to analyze the data and answer the questions:part a: what percentage of the survey respondents do not like both hamburgers and burritos? (2 points)part b: what is the marginal relative frequency of all customers that like hamburgers? (3 points)part c: use the conditional relative frequencies to determine which data point has strongest association of its two factors. use complete sentences to explain your answer. (5 points)will mark brainliest if correct answers Avery and his coworkers showed that the agent responsible for changing nonvirulent bacteria into virulent bacteria was What is a substrate A cylindrical metal rod has a resistance R . If both its length and its diameter are tripled, its new resistance will be: Using the following image, find QR given RS=6 and QS=7. What is QR? The current in a wire is 3 a . calculate the charge that flows through the wire in 1 minute Determine which function has a greater initial value and explain how you know.Function 1: y = 2()*Function 2:xy03162 123 24 What volume of water vapor would be produced from the combustion of 815.74 grams of propane (C3H8) with 1,006.29 grams of oxygen gas, under a pressure of 1.05 atm and a temperature of 350. degrees C? Given:C3H8(g) + 5 O2(g) ---> 3 CO2(g) + 4 H2O(g)(OR C3 H8 ("g") + 5 O2 ("g") right arrow 3 C O2 ("g") + 4 H2O ("g")Do not type units with your answer james is a wells fargo employee. his manager saw him parking illegally in the loading zone once, so he assumes james must have engaged in fraudulent account activity. which of the following perceptual errors did his manager make?